Time Dilation Formula: Clarifying Confusion

In summary: For example, if the "moving" clock is moving close to the speed of light, then relativistic effects will cause the time measured by a stationary observer to be "slowed down" by a factor proportional to the speed of the moving clock.
  • #106
Ich said:
It isn't traveling the same distance.

But it is, within the clock's frame of reference, and the speed of light has to be the same wherever it is observed from
 
Physics news on Phys.org
  • #107
Ich said:
I still don't see it. The problem Einstein refers to arises if you say that w=c-v is the speed of light as measured by the carriage. It isn't.

Einstein wrote:
w is the required velocity of light with respect to the carriage, and we have
w = c - v.
The velocity of propagation of a ray of light relative to the carriage thus comes out smaller than c.
 
  • #108
Ich said:
They didn't write what you read. There is no such thing as "observing the clock's frame of reference". You may observe the clock, recording time and date of such measurements as given by your frame of reference.


Einstein wrote in http://www.bartleby.com/173/11.html"
How are we to find the place and time of an event in relation to the train, when we know the place and time of the event with respect to the railway embankment? Is there a thinkable answer to this question of such a nature that the law of transmission of light in vacuo does not contradict the principle of relativity? In other words: Can we conceive of a relation between place and time of the individual events relative to both reference-bodies, such that every ray of light possesses the velocity of transmission c relative to the embankment and relative to the train? This question leads to a quite definite positive answer, and to a perfectly definite transformation law for the space-time magnitudes of an event when changing over from one body of reference to another.

So I read this as Einstein being concerned about finding the space-time co-ordinates from one frame of reference and using them (transforming them) to find the corresponding space-time magnitudes within another frame of reference.

Why don't you read it as saying:
"However to an observer who sees the clock pass at velocity v, the light takes more time to traverse the length of the clock when the pulse is traveling in the same direction as the clock, and it takes less time for the return trip." ?

And that is exactly how Galileo or Newton would have read it!

But we are not concerned with a Galilean transformation.

Again: why don't you try to understand what they wrote instead of interpreting their every word? That has nothing to do with accusing you of being dim, but you definitely always skip the first step - reading and following their argument - and jump to the second: interpreting hat you have learned.

I'm sorry if I have given that impression; I have read and understood exactly what they are saying and I am, I suppose, guilty of trying to re-interpret it in relation to SR.

I think a reasonable first step would be if you draw a spacetime diagram of the described events; you can then easily read off distances, times, and velocities, and check with what the link is saying contrary to what you have assumed it was saying.


Please allow me to do just what you say and draw some diagrams and then check back with you with my interpretaions of them!

Grimble:smile:

ps thank you for your time!
 
Last edited by a moderator:
  • #109
Hello Grimble.

As you say :-
Einstein wrote:

w is the required velocity of light with respect to the carriage, and we have
w = c - v.
The velocity of propagation of a ray of light relative to the carriage thus comes out smaller than c.


But if you follow the argument from beginning to end he explains that this relation cannot hold unless we abandon the principle of relativity, and as we know, he did not abandon it.

I have not had time to reread the translation of Einstein's work, but I recall that it is probably one of the best and simplest explantions of the theory. I think it may be unwise to put a different interpretation on what Einstein wrote, after all its his theory and subsequent events have so far proved him correct. He proposed the constancy of c in inertial frames and so how likely do you think it is that he would argue against it except to make a point of how the theory would not hold up if this proposition was untrue.

Matheinste.
 
  • #110
Hello again Matheinste!:smile::smile:
matheinste said:
Hello Grimble.

As you say :-
Einstein wrote:

w is the required velocity of light with respect to the carriage, and we have
w = c - v.
The velocity of propagation of a ray of light relative to the carriage thus comes out smaller than c.


But if you follow the argument from beginning to end he explains that this relation cannot hold unless we abandon the principle of relativity, and as we know, he did not abandon it.

I have not had time to reread the translation of Einstein's work, but I recall that it is probably one of the best and simplest explantions of the theory. I think it may be unwise to put a different interpretation on what Einstein wrote, after all its his theory and subsequent events have so far proved him correct. He proposed the constancy of c in inertial frames and so how likely do you think it is that he would argue against it except to make a point of how the theory would not hold up if this proposition was untrue.

Matheinste.

I agree with everything you say here! c is constant and the same wherever we measure it and that quote would deny relativity!

I was quoting Einstein in response to an earlier post by Ich who stated that:
The problem Einstein refers to arises if you say that w=c-v is the speed of light as measured by the carriage. It isn't.

This is indeed the simplest and most elegant explanation of his theory and, to me at least, it makes perfect sense and agrees with everything one tries to put into space-time diagrams.
Particular favourites of mine are diagrams of Minkowski space, particularly the traditional Minkowski Diagrams elevated from flat two dimensional diagrams into 3 dimensional constructs which treatment is what I believe really shows everything clearly.
 
  • #111
Grimble said:
Yes, I can see what you are saying BUT you have neglected to say that the 8 light-seconds is measured in co-ordinate seconds, not proper seconds and how do you apply Euclidean/Newtonian physics to a rectangle whose sides are measured in different units?
Both sides of the moving rectangle are measured in coordinate light-seconds. When you want to compute a distance in any coordinate system, you only use that coordinate system's measures of distance. Again, if you want to know the distance between two points in a given frame, you take the coordinate distance between them on the x-axis which we can call dx, the coordinate distance on the y-axis dy, and the coordinate distance on the z-axis dz, then use the pythagorean formula [tex]\sqrt{dx^2 + dy^2 + dz^2}[/tex] to get the total distance between the points. The procedure is the same regardless of whether you are measuring the distance between objects (or parts of an object, like different corners of a square) that are at rest in this frame, or between objects that are in motion in this frame. Nowhere would the notion of "proper distance" enter into it at all.
JesseM said:
I don't understand, why do you think the 1st postulate implies that the time should be constant? The 1st postulate implies that if you run the same experiment in different frames each frame will see the same result, so if you construct a light clock at rest in frame A and measures the time in frame A, it should give the same answer that you'd get if you constructed an identical light clock at rest in frame B and measured the time in frame B. But the 1st postulate does not say that if you construct a light clock at rest in frame A but in motion in frame B, and measure the time in frame B, that the result would be the same as if you construct a clock at rest in frame B and measure the time in frame B. In this case we are dealing with two clocks that have different velocities in frame B, but we are measuring both their ticking rates from the perspective of frame B--nothing about the 1st postulate suggests that their ticking rates should be identical.
Grimble said:
You are quite right when you say that the two clocks should keep identical time;
Identical time in their own respective rest frames. They certainly do not keep identical time if you measure both from the perspective of a single inertial frame in which they have different speeds--that's exactly what I meant when I said above "In this case we are dealing with two clocks that have different velocities in frame B, but we are measuring both their ticking rates from the perspective of frame B--nothing about the 1st postulate suggests that their ticking rates should be identical." Do you think the 1st postulate implies their ticking rate should be identical even in this situation? If so, can you explain your reasoning?
Grimble said:
if the moving clock were to transmit a pulse of light each second, then allowing for the transmission time for the pulse of light, the stationary frame would still determine that the moving clock was 'ticking' at the same rate as its own clock.
No, it definitely would not! Time dilation is what remains after you correct for transmission delays (i.e. correcting for the Doppler effect). For example, suppose the clock is moving at 0.6c, and in 2020 I see it next to a marker 10-light-years away from me (in my frame) with the clock showing a reading of 30 years, and then in 2036 I see it next to a marker 16-light-years-away from me with the clock showing a reading of 38 years. If I subtract off the light travel times (10 years for the first reading to travel 10 light-years from the clock to my eyes, and 16 years for the second reading to travel 16 light-years from the clock to my eyes), I will conclude that the clock "really" showed the first reading of 30 years in 2020-10=2010, and it "really" showed the second reading of 38 years in 2036-16=2020. So I will conclude that in the 10 years between 2010 and 2020, the clock itself only ticked forward by 8 years from 30 to 38, so it must have been slowed down by a factor of 0.8 in my frame. This is different from how much it appeared to be slowed down visually--visually it took 2036-2020=16 years to tick forward by 8 years, so it appeared to be running slow by a factor of 0.5, but this extra slowdown is just due to the Doppler effect (which is a consequence of the fact that light from different readings on the clock has different delays in reaching me since the clock's distance from me is changing).
JesseM said:
I'm not sure what you mean by "space and time not being absolute"--certainly it's true that the distance and time between a pair of events will vary depending on what frame you use, but there is nothing in SR that implies we can't use the Euclidean formula for the distance between events in a single frame, or that we can't use the ordinary kinematical formula that velocity = distance/time in each frame. The light clock thought-experiment makes use of these ordinary geometrical and kinematical rules which still apply in relativity, along with the uniquely relativistic notion that if the light is bouncing between mirrors at c in the light clock's own rest frame, it must also be bouncing between them at c in the frame where the light clock is in motion.
Grimble said:
I am referring to Einstein's statement in http://www.bartleby.com/173/11.html" where he says
THE RESULTS of the last three sections show that the apparent incompatibility of the law of propagation of light with the principle of relativity (Section VII) has been derived by means of a consideration which borrowed two unjustifiable hypotheses from classical mechanics; these are as follows:

1. The time-interval (time) between two events is independent of the condition of motion of the body of reference.
2. The space-interval (distance) between two points of a rigid body is independent of the condition of motion of the body of reference.
OK, but in that quote he was saying exactly the same thing I was saying when I said "certainly it's true that the distance and time between a pair of events will vary depending on what frame you use". Do you agree that there is no conflict between this statement and my other statement immediately after? Namely:
there is nothing in SR that implies we can't use the Euclidean formula for the distance between events in a single frame, or that we can't use the ordinary kinematical formula that velocity = distance/time in each frame. The light clock thought-experiment makes use of these ordinary geometrical and kinematical rules which still apply in relativity, along with the uniquely relativistic notion that if the light is bouncing between mirrors at c in the light clock's own rest frame, it must also be bouncing between them at c in the frame where the light clock is in motion.
 
Last edited by a moderator:
  • #112
JesseM said:
Both sides of the moving rectangle are measured in coordinate light-seconds.

I am afraid you have lost me here with yor reasoning, JesseM; for if the two sides parallel to the direction of motion have been transformed by the Lorentz factor and the other two sides are unchanged, how can they be in the same units? The Lorentz transformations change the size of the units as well as their quantity or are you saying all that was discussed previously in this thread is nonsense?:smile:
 
  • #113
JesseM said:
Identical time in their own respective rest frames. They certainly do not keep identical time if you measure both from the perspective of a single inertial frame in which they have different speeds--that's exactly what I meant when I said above "In this case we are dealing with two clocks that have different velocities in frame B, but we are measuring both their ticking rates from the perspective of frame B--nothing about the 1st postulate suggests that their ticking rates should be identical." Do you think the 1st postulate implies their ticking rate should be identical even in this situation? If so, can you explain your reasoning?
Yes, my reasoning is to follow what the 1st postulate says:
The Principle of Relativity – The laws by which the states of physical systems undergo change are not affected, whether these changes of state be referred to the one or the other of two systems in uniform translatory motion relative to each other.
or as he says in http://www.bartleby.com/173/5.html"
If K is a Galileian co-ordinate system, then every other co-ordinate system K' is a Galileian one, when, in relation to K, it is in a condition of uniform motion of translation. Relative to K' the mechanical laws of Galilei-Newton hold good exactly as they do with respect to K. 2
We advance a step farther in our generalisation when we express the tenet thus: If, relative to K, K' is a uniformly moving co-ordinate system devoid of rotation, then natural phenomena run their course with respect to K' according to exactly the same general laws as with respect to K. This statement is called the principle of relativity (in the restricted sense).

And if, natural phenomena run their course with respect to K' according to exactly the same general laws as with respect to K, then time and distance must be identical. (A muon's half life cannot be different and if time is identical distance has to be also cf. the speed of light)

That is; they will keep the same 'Proper Time' within their own frames of reference.

So in two independent Inertial Frames of Reference, identical clocks will keep identical time.

But yes, if viewed by an independent observer they will shew different times.

Grimble:smile:
 
Last edited by a moderator:
  • #114
JesseM said:
OK, but in that quote he was saying exactly the same thing I was saying when I said "certainly it's true that the distance and time between a pair of events will vary depending on what frame you use". Do you agree that there is no conflict between this statement and my other statement immediately after? Namely:
there is nothing in SR that implies we can't use the Euclidean formula for the distance between events in a single frame, or that we can't use the ordinary kinematical formula that velocity = distance/time in each frame. The light clock thought-experiment makes use of these ordinary geometrical and kinematical rules which still apply in relativity, along with the uniquely relativistic notion that if the light is bouncing between mirrors at c in the light clock's own rest frame, it must also be bouncing between them at c in the frame where the light clock is in motion.

No, because
The light clock thought-experiment makes use of these ordinary geometrical and kinematical rules which still apply in relativity
but it applies them across frames.
 
  • #115
JesseM said:
No, it definitely would not! Time dilation is what remains after you correct for transmission delays (i.e. correcting for the Doppler effect). For example, suppose the clock is moving at 0.6c, and in 2020 I see it next to a marker 10-light-years away from me (in my frame) with the clock showing a reading of 30 years, and then in 2036 I see it next to a marker 16-light-years-away from me with the clock showing a reading of 38 years. If I subtract off the light travel times (10 years for the first reading to travel 10 light-years from the clock to my eyes, and 16 years for the second reading to travel 16 light-years from the clock to my eyes), I will conclude that the clock "really" showed the first reading of 30 years in 2020-10=2010, and it "really" showed the second reading of 38 years in 2036-16=2020. So I will conclude that in the 10 years between 2010 and 2020, the clock itself only ticked forward by 8 years from 30 to 38, so it must have been slowed down by a factor of 0.8 in my frame. This is different from how much it appeared to be slowed down visually--visually it took 2036-2020=16 years to tick forward by 8 years, so it appeared to be running slow by a factor of 0.5, but this extra slowdown is just due to the Doppler effect (which is a consequence of the fact that light from different readings on the clock has different delays in reaching me since the clock's distance from me is changing).

As I said earlier, identical clocks in Inertial frames of reference will keep identical time.
It is only when one is observed from the other that time dilation occurrs.
Time dilation is the phenomenon where the time observed from one frame is different from that observed from the other.
So the observer from the other frame will see the time transformed in unit size and number of units but the total duration in absolute terms has to be the same - the half-life of the muon cannot change, only how it is measured can, as demonstrated by the afore mentioned experiment where the half-life was extended to 65secs, 65 transformed seconds that are [itex]\frac{1}{29.4}[/itex] of the laboratory seconds.

cf.
matheinste said:
With regards to the first point the wording seems over complicated and still confuses me. Clocks just show time

Perhaps I can give examples, in my view, of faulty and correct reasoning with regard to the often used example of the muon's lifetime as an aid to illustrating time dialtion. These two methods lead to exactly the opposite outcome.

Let the lab frame be regarded as the stationary frame and the muon's frame the moving frame with repect to it. We can use the values of 2 microseconds 60 microseconds as being the figures used for the decay times of the muon measured by clocks in the muon and lab frame respectivley. Both explanations are non rigorous.

WRONG reasoning:- The muon's lifetime of 2 micoseconds its own frame is extended to 60 microseconds in the lab frame. This is an example of time dilation this shows that the number of seconds which the muon lives is dilated, made bigger, to 60 microseconds.

Now bear in mind the definition which says that a moving clock viewed from a stationary frame runs slow, and reason as follows.

CORRECT reasoning:- The muon has a lifetime as measured in its own frame, the time measured by a clock carried with it, its proper time, of 2 microseconds. This is an invariant and is the same for everyone, it cannot be changed. In the lab frame this is measured as 60 microseconds. This is an example of time dilation and shows that 2 microseconds in the muon's frame takes 60 microseconds to pass in the lab frame. So the time it takes 2 microseconds to pass in the moving frame is dilated to 60 microseconds as viewed from the stationary frame. That is, the preiod is extended.

Remember that although the lab frame measures 60 microseconds, the lab observers still agree that the muon's clock reads a proper, invariant time of 2 microseconds.

Matheinste.
 
  • #116
Grimble said:
JesseM said:
Identical time in their own respective rest frames. They certainly do not keep identical time if you measure both from the perspective of a single inertial frame in which they have different speeds--that's exactly what I meant when I said above "In this case we are dealing with two clocks that have different velocities in frame B, but we are measuring both their ticking rates from the perspective of frame B--nothing about the 1st postulate suggests that their ticking rates should be identical." Do you think the 1st postulate implies their ticking rate should be identical even in this situation? If so, can you explain your reasoning?
And if, natural phenomena run their course with respect to K' according to exactly the same general laws as with respect to K, then time and distance must be identical. (A muon's half life cannot be different and if time is identical distance has to be also cf. the speed of light)

That is; they will keep the same 'Proper Time' within their own frames of reference.

So in two independent Inertial Frames of Reference, identical clocks will keep identical time.

But yes, if viewed by an independent observer they will shew different times."

Grimble:smile:
You bolded the second half of my sentence but then ignored the first half, taking the meaning out of context. I first said "In this case we are dealing with two clocks that have different velocities in frame B, but we are measuring both their ticking rates from the perspective of frame B"--so when I then said "nothing about the 1st postulate suggests that their ticking rates should be identical", I was clearly talking about their ticking rates in frame B, not their ticking rates in their own respective rest frames. Hopefully you'd agree that nothing about the first postulate suggests that their ticking rates should be identical in frame B, given that one is at rest in frame B and the other is not?
 
  • #117
Grimble said:
JesseM said:
The light clock thought-experiment makes use of these ordinary geometrical and kinematical rules which still apply in relativity
No, because but it applies them across frames.
How do you think the light clock thought-experiment applies these rules "across frames"? The light clock thought-experiment derives the slowed down rate of ticking of the moving light clock using only a single frame, namely the frame in which the light clock is moving--the derivation only uses velocities and distances and times which are measured in the coordinates of that frame.

Do you understand that just because I am observing a clock which is moving relative to myself, does not mean that I need to use any frames other than my own rest frame to analyze its behavior? That talking about the properties of an object which is moving relative to me (like the time on a moving clock) does not in any way imply I am comparing multiple frames, I can analyze these properties just fine using nothing but my own rest frame? A frame is just a coordinate system after all, I can perfectly well keep track of the way the position coordinate of the moving object changes with coordinate time using just the coordinates of my rest frame.

If you think any frames other than the observer's rest frame are used in analyzing the light clock, can you point out the specific step in the analysis where you think this happens? For example, do we need any frames other than the observer's frame to figure out the distance the mirrors have traveled horizontally in a given time if we know their velocity v? Do we need any frames other than the observer's frame to use the pythagorean theorem to figure out the diagonal distance the light must travel if we know the horizontal distance traveled by the mirrors (just v*t, where t is the time between the light hitting the top and bottom mirror and v is the horizontal velocity of the mirrors) and the vertical distance h between them? Do we need any frames other than the observer's frame to figure out the time T that would be required in order to ensure that the diagonal distance D = [tex]\sqrt{v^2 T^2 + h^2}[/tex] will satisfy D/T = c? (making use of the second postulate which says light must move at c in every frame, including the observer's frame, along with the ordinary kinematical rule that speed = distance/time)

From all this, you can conclude that the time T between ticks of the light clock in the observer's frame must be equal to [tex]T = \frac{h}{\sqrt{c^2 - v^2}}[/tex]. Only here do you have to consider another frame if you want to derive the time dilation formula from this--you have to figure out what the time t between ticks would be in the light clock's own rest frame, and obviously if h is the vertical distance between mirrors this would be t = h/c (here you do need to make an argument to show the vertical height h will be the same in both frames, that there will be no length contraction perpendicular to the direction of motion). Then if you divide T/t you get the gamma factor [tex]\frac{1}{\sqrt{1 - v^2/c^2}}[/tex]. But this is just simple division, when deriving the time between ticks in each frame you can work exclusively with the coordinates of that frame and not worry about other frames.
 
Last edited:
  • #118
Grimble said:
As I said earlier, identical clocks in Inertial frames of reference will keep identical time.
It is only when one is observed from the other that time dilation occurrs.
Time dilation is the phenomenon where the time observed from one frame is different from that observed from the other.
So the observer from the other frame will see the time transformed in unit size and number of units but the total duration in absolute terms has to be the same - the half-life of the muon cannot change, only how it is measured can, as demonstrated by the afore mentioned experiment where the half-life was extended to 65secs, 65 transformed seconds that are [itex]\frac{1}{29.4}[/itex] of the laboratory seconds.
I'm not sure what you mean "the total duration in absolute terms". Certainly if you're talking about "proper time", meaning the time as measured by a clock moving along with the object (in this case the muon), then it is true that there is no disagreement between frames about the proper time between two events on the object's worldline (like the muon being created and then decaying, which you can average for many muons to derive the half-life). But what does this point about proper times have to do with the light clock derivation of the time dilation equation, an equation which deals with coordinate time in the frame where the light clock is moving, not proper time?

Also, you can see that time dilation is not just a sort of illusion created by using coordinate time rather than proper time by considering a case where two clocks depart from the same location and then later return to a common location, as in the twin paradox--in this case one clock may actually have elapsed less proper time (aged less) than the other. And you can calculate how much proper time each elapsed if you know the coordinate times t0 and t1 of the first and second meetings of the two clocks in some inertial frame, and you know a given clock's velocity as a function of time v(t) in that frame...then you can take the time dilation equation [tex]dT = \sqrt{1 - v^2/c^2} * dt[/tex] (where dT is the proper time and dt is the coordinate time) and integrate it to find the total proper time elapsed on the clock, i.e. [tex]\int_{t_0}^{t_1} \sqrt{1 - v(t)^2/c^2} \, dt[/tex] integrated over the coordinate time t. So you can see that even if your ultimate interest is in knowing the proper time between two events on an object's worldline, the coordinate time between the events in some inertial frame, along with the time dilation equation and the object's velocity as a function of time in that frame, can be used to calculate the proper time.
 
Last edited:
  • #119
Sorry, I missed a few posts of yours.
Ich said:
It isn't traveling the same distance.

But it is, within the clock's frame of reference, and the speed of light has to be the same wherever it is observed from
From this, and some of your comments before, it is obvious that you have no idea what a reference frame is good for, or even how "speed" is defined. You should get familiar with this basic stuff in relativity before you move on.
So I invite you to draw that spacetime diagram of the quoted situation, and post it here along with the derivation of light travel time and speed of light. You'll encounter some points where you don't know how to proceed; it would be most fruitful if we could help you exactly with these points.
 
  • #120
Ich said:
Sorry, I missed a few posts of yours.

From this, and some of your comments before, it is obvious that you have no idea what a reference frame is good for, or even how "speed" is defined. You should get familiar with this basic stuff in relativity before you move on.
So I invite you to draw that spacetime diagram of the quoted situation, and post it here along with the derivation of light travel time and speed of light. You'll encounter some points where you don't know how to proceed; it would be most fruitful if we could help you exactly with these points.

I am sorry Ich, that I seem to be getting lost about what I am doing:redface: and I will draw some diagrams that we can discuss, just give me a little time.

I have been picturing the stationary clock as being placed on Einstein's embankment and the moving clock riding on his train; am I misreading this situation?

As for speed, I take that as non-directional (for speed with direction is velocity?) and it is distance/time.

And thank you for your offer of help:smile:
 
  • #121
JesseM said:
How do you think the light clock thought-experiment applies these rules "across frames"? The light clock thought-experiment derives the slowed down rate of ticking of the moving light clock using only a single frame, namely the frame in which the light clock is moving--the derivation only uses velocities and distances and times which are measured in the coordinates of that frame.

Do you understand that just because I am observing a clock which is moving relative to myself, does not mean that I need to use any frames other than my own rest frame to analyze its behavior? That talking about the properties of an object which is moving relative to me (like the time on a moving clock) does not in any way imply I am comparing multiple frames, I can analyze these properties just fine using nothing but my own rest frame? A frame is just a coordinate system after all, I can perfectly well keep track of the way the position coordinate of the moving object changes with coordinate time using just the coordinates of my rest frame.

I'm sorry if I am getting confused here, but as I have just said in my last post I was visualising this as Einstein's embankment and moving train where he thought it necessary to use separate co-ordinate frames. Was he over complicating it when he could have worked it all in relation to the embankment, is that what you are saying? Or am I becoming confused again?

It does seem to be complicated and confusing, when discussing these things, to agree on what we are intending to convey. Oh the complexities of language and syntax!

Grimble:wink:
 
  • #122
JesseM said:
You bolded the second half of my sentence but then ignored the first half, taking the meaning out of context. I first said "In this case we are dealing with two clocks that have different velocities in frame B, but we are measuring both their ticking rates from the perspective of frame B"--so when I then said "nothing about the 1st postulate suggests that their ticking rates should be identical", I was clearly talking about their ticking rates in frame B, not their ticking rates in their own respective rest frames. Hopefully you'd agree that nothing about the first postulate suggests that their ticking rates should be identical in frame B, given that one is at rest in frame B and the other is not?

Mea Culpa! once again.

The bolding in the above quoted post was unintentional, and I hadn't realized it had happened until you pointed it out; it was completely unintentional and changed the whole meaning of my reply SORRY!

I do certainly agree that they would have different rates of ticking when measured from the same frame.

Grimble:redface::redface::redface:
 
  • #123
I have been picturing the stationary clock as being placed on Einstein's embankment and the moving clock riding on his train; am I misreading this situation?
Didn't you talk abouthttp://www.pa.msu.edu/courses/2000spring/PHY232/lectures/relativity/contraction.html" ?
As for speed, I take that as non-directional (for speed with direction is velocity?) and it is distance/time.
Yes, distance/time if both are measured in the same reference frame. See the difference to your statements?
Things should become a lot clearer if you know and use the definition of the standard terms. Carry on with the diagrams.
 
Last edited by a moderator:
  • #124
Grimble said:
I'm sorry if I am getting confused here, but as I have just said in my last post I was visualising this as Einstein's embankment and moving train where he thought it necessary to use separate co-ordinate frames. Was he over complicating it when he could have worked it all in relation to the embankment, is that what you are saying? Or am I becoming confused again?
Einstein wasn't analyzing time dilation in the train/embankment thought-experiment, he was analyzing the relativity of simultaneity, and since the relativity of simultaneity is all about how simultaneity differs between two frames of course he needed to look at the thought-experiment from the perspective of both frames. But that doesn't mean every analysis of a moving object requires multiple frames.

Again, do you understand the difference between talking about an object moving relative to you and talking about a frame moving relative to you? If I am at rest on the embankment, I can perfectly well analyze the behavior of a moving train, or a moving light clock aboard the train, using only the embankment frame, without ever making reference to the train's own rest frame--do you disagree with that? If not, note that this is in fact the sort of thing I did with the moving light clock, figuring out the time between ticks using only the observer's frame, not the light clock's own rest frame. And if you do disagree, please address this previous post:
JesseM said:
If you think any frames other than the observer's rest frame are used in analyzing the light clock, can you point out the specific step in the analysis where you think this happens? For example, do we need any frames other than the observer's frame to figure out the distance the mirrors have traveled horizontally in a given time if we know their velocity v? Do we need any frames other than the observer's frame to use the pythagorean theorem to figure out the diagonal distance the light must travel if we know the horizontal distance traveled by the mirrors (just v*t, where t is the time between the light hitting the top and bottom mirror and v is the horizontal velocity of the mirrors) and the vertical distance h between them? Do we need any frames other than the observer's frame to figure out the time T that would be required in order to ensure that the diagonal distance D = [tex]\sqrt{v^2 T^2 + h^2}[/tex] will satisfy D/T = c? (making use of the second postulate which says light must move at c in every frame, including the observer's frame, along with the ordinary kinematical rule that speed = distance/time)

From all this, you can conclude that the time T between ticks of the light clock in the observer's frame must be equal to [tex]T = \frac{h}{\sqrt{c^2 - v^2}}[/tex]. Only here do you have to consider another frame if you want to derive the time dilation formula from this--you have to figure out what the time t between ticks would be in the light clock's own rest frame, and obviously if h is the vertical distance between mirrors this would be t = h/c (here you do need to make an argument to show the vertical height h will be the same in both frames, that there will be no length contraction perpendicular to the direction of motion). Then if you divide T/t you get the gamma factor [tex]\frac{1}{\sqrt{1 - v^2/c^2}}[/tex]. But this is just simple division, when deriving the time between ticks in each frame you can work exclusively with the coordinates of that frame and not worry about other frames.
 
Last edited:
  • #125
Ich said:
Didn't you talk abouthttp://www.pa.msu.edu/courses/2000spring/PHY232/lectures/relativity/contraction.html" ?

Yes, but surely this is the equivalent of http://www.bartleby.com/173/7.html" where Einstein writes:
The velocity W of the man relative to the embankment is here replaced by the velocity of light relative to the embankment. w is the required velocity of light with respect to the carriage, and we have
w = c - v.
The velocity of propagation of a ray of light relative to the carriage thus comes out smaller than c.
But this result comes into conflict with the principle of relativity set forth in Section V. For, like every other general law of nature, the law of the transmission of light in vacuo must, according to the principle of relativity, be the same for the railway carriage as reference-body as when the rails are the body of reference. But, from our above consideration, this would appear to be impossible. If every ray of light is propagated relative to the embankment with the velocity c, then for this reason it would appear that another law of propagation of light must necessarily hold with respect to the carriage—a result contradictory to the principle of relativity.

And don't worry I am continuing with my diagrams. (This is all fascinating!):smile::smile:
 
Last edited by a moderator:
  • #126
JesseM said:
Again, do you understand the difference between talking about an object moving relative to you and talking about a frame moving relative to you? If I am at rest on the embankment, I can perfectly well analyze the behavior of a moving train, or a moving light clock aboard the train, using only the embankment frame, without ever making reference to the train's own rest frame--do you disagree with that?

Not at all,

If not, note that this is in fact the sort of thing I did with the moving light clock, figuring out the time between ticks using only the observer's frame, not the light clock's own rest frame.

as indeed Galileo and Newton would have done! :approve: But what has it to do with SR? :smile:

Let me address the previous post you quote.

We have the horizontal distance = vT,
The vertical distance h (which, if the clock is ticking seconds in this frame = 1 x c)
and the diagonal distance D, where [tex]D = \sqrt{{v^2}{T^2} + {h^2}}[/tex]

And you quite rightly say that only one frame is needed to reach this point. In fact it could be done using either frame.
Consider, if you will, that if we were to use the light clock's own frame of reference, then we should still say that the oberver, traveling at v relative to the clock would still see the diagonal [tex]D = \sqrt{{v^2}{T^2} + {h^2}}[/tex].

I have no problem with this at all, nor with the conclusion that:
T is the time for light to travel from the mirror to the observer while
t is the time for the light to travel back to the source
and that if the clock is ticking seconds in its own frame of reference then it will take γ seconds, measured in that same frame, to reach the observer.

But the first postulate also requires that an identical clock in the oberver's frame of reference would also be ticking seconds, identical seconds as they are both inertial frames of reference, or Galilean frames as Einstein termed them.

What we have to determine here, is how to resolve the difference in what the observer sees, between the time in his own frame of 1 second and that he observes in the clock's frame of γ seconds.

Thankfully Einstein gave us the way of doing this:
We transform the time observed in the clock's frame using the Lorentz Transformation, and thus we find that T, in transformed units( in order to avoid any dispute over exactly what we are referring to I shall refer to them as transformed units and inertial units) is equal to 1 second inertial time.

But if T inertial seconds = γ seconds in (inertial) time, then

T in transformed seconds = γ seconds in transformed time
yet
T in transformed seconds = 1 second in inertial time
therefore
γ seconds in transformed time = 1 second in inertial time

or in the more usual terms used [tex]{t^'} = \frac{t}{\gamma}[/tex] which is what we should expect as it reflects the analagous formula for length contraction [tex]{L^'} = \frac{L}{\gamma}[/tex] from which we can see that if [tex]c = \frac{L}{t}[/tex] then [tex]c = \frac{L^'}{t^'}[/tex] which of course it has to do.

Now one last little consideration:

If we say that v = 0.866c then [itex] \gamma = 2 [/itex] and if we apply this to our scenario above
we find that the time for light to traverse the diagonal path = [itex]\gamma[/itex] = 2 seconds inertial time = [itex]2\gamma[/itex] (or [itex]{\gamma^2}[/itex]) [itex] = 4 [/itex]seconds transformed time
and the corresponding diagonal distance will be 4 transformed light seconds
so the observer, observing the clocks reference frame will see the light take 4 transformed seconds to cover the 4 transformed light seconds; that is the equivalent of 2 seconds to cover 2 lightseconds inertial time. (all at the speed of c)

And if you have followed all my ramblings you can see that everything adds up and matches up.
I.e. the clock ticks once a second for both the observer and the clock. In the clocks frame of reference the light will take two seconds to reach the observer, but to the observer, observing the clocks frame of reference it will take 4 transformed seconds!

All as neat and tidy as Einstein could have wished.:smile::smile::smile:
 
  • #127
Grimble said:
as indeed Galileo and Newton would have done! :approve: But what has it to do with SR? :smile:
Galileo or Newton would not have assumed that the light must be traveling at c in both their own frame and the clock's rest frame--in fact they would have assumed that if it is traveling at c in one of those frames, it must be traveling at a different speed in the other frame, in such a way that both frames end up agreeing on the time between ticks.
Grimble said:
Let me address the previous post you quote.

We have the horizontal distance = vT,
The vertical distance h (which, if the clock is ticking seconds in this frame = 1 x c)
and the diagonal distance D, where [tex]D = \sqrt{{v^2}{T^2} + {h^2}}[/tex]

And you quite rightly say that only one frame is needed to reach this point. In fact it could be done using either frame.
Consider, if you will, that if we were to use the light clock's own frame of reference, then we should still say that the oberver, traveling at v relative to the clock would still see the diagonal [tex]D = \sqrt{{v^2}{T^2} + {h^2}}[/tex].

I have no problem with this at all, nor with the conclusion that:
T is the time for light to travel from the mirror to the observer while
t is the time for the light to travel back to the source
and that if the clock is ticking seconds in its own frame of reference then it will take γ seconds, measured in that same frame, to reach the observer.
OK, so we agree that the time between ticks for the moving light clock can be derived using only the observer's frame. Then as I pointed out in that earlier post, if you want to derive the time dilation equation which compares time in the clock's rest frame to time in the observer's frame, that's when you do have to bring in some assumptions about multiple frames:
JesseM said:
From all this, you can conclude that the time T between ticks of the light clock in the observer's frame must be equal to [tex]T = \frac{h}{\sqrt{c^2 - v^2}}[/tex]. Only here do you have to consider another frame if you want to derive the time dilation formula from this--you have to figure out what the time t between ticks would be in the light clock's own rest frame, and obviously if h is the vertical distance between mirrors this would be t = h/c (here you do need to make an argument to show the vertical height h will be the same in both frames, that there will be no length contraction perpendicular to the direction of motion). Then if you divide T/t you get the gamma factor [tex]\frac{1}{\sqrt{1 - v^2/c^2}}[/tex]. But this is just simple division, when deriving the time between ticks in each frame you can work exclusively with the coordinates of that frame and not worry about other frames.
The assumption that the vertical height between mirrors is the same in both frames does involve thinking about multiple frames in SR, as does the assumption that if the height is h in the clock's rest frame, the time between ticks must be t = h/c in that frame.
Grimble said:
What we have to determine here, is how to resolve the difference in what the observer sees, between the time in his own frame of 1 second and that he observes in the clock's frame of γ seconds.
You have it backwards here. The time in the observer's frame will be the greater time, not the lesser time (and the gamma factor γ is always greater than 1). So, if the time in the clock's frame is 1 second, the time in the observer's frame will be gamma seconds (and if the time in the clock's frame is T seconds, the time in the observer's frame is T*gamma seconds)
Grimble said:
We transform the time observed in the clock's frame using the Lorentz Transformation, and thus we find that T, in transformed units( in order to avoid any dispute over exactly what we are referring to I shall refer to them as transformed units and inertial units) is equal to 1 second inertial time.
That's not good terminology, since both frames are "inertial" ones in the terminology of relativity. Also it's not as if the time in the observer's frame is intrinsically the one that's been "transformed", you can equally well start out with the time in the observer's frame and then use the Lorentz transformation to derive the time in the clock's frame. Better terminology would just be to give names to the two frames, like "clock's frame" and "observer's frame", or just use different notation to refer to them like unprimed t vs. primed t'.
Grimble said:
But if T inertial seconds = γ seconds in (inertial) time, then
What? I thought you were using "inertial" time to refer to time in the clock's frame, but the time in the clock's frame cannot be both T seconds and gamma seconds. If the time between ticks of the clock in the clock's frame is T seconds, then the time between ticks of that same clock in the observer's frame (what you were calling 'transformed' time) would be gamma*T seconds.
Grimble said:
T in transformed seconds = γ seconds in transformed time
yet
T in transformed seconds = 1 second in inertial time
therefore
γ seconds in transformed time = 1 second in inertial time
Don't understand these either. And why are you using three times--1 second, T seconds, and gamma seconds? If we have only two frames to consider there should be only two times involved. If the time in the clock's frame is 1 second than the time in the observer's frame will be gamma seconds, while if we say more generally that the time in the clock's frame is T seconds (i.e. not assuming the distance between mirrors is 1 light-second), then the time in the observer's frame is gamma*T seconds.
Grimble said:
or in the more usual terms used [tex]{t^'} = \frac{t}{\gamma}[/tex]
If t is supposed to be the time in the clock's own rest frame and t' is the time in the observer's frame, then this formula is wrong, you should be multiplying by gamma rather than dividing by it: [tex]t' = t * \gamma[/tex].
Grimble said:
which is what we should expect as it reflects the analagous formula for length contraction [tex]{L^'} = \frac{L}{\gamma}[/tex]
This is the correct formula for length contraction if L is the object's length in its own rest frame and L' is the length in the observer's frame, but you can see that it's not exactly analogous to the correct formula for time dilation I wrote above, the formula for time dilation involves multiplying by gamma while the formula for length contraction involves dividing by gamma.
Grimble said:
from which we can see that if [tex]c = \frac{L}{t}[/tex] then [tex]c = \frac{L^'}{t^'}[/tex] which of course it has to do.
Nope, the fact that the speed of light is c in both frames cannot be derived from the length contraction and time dilation formulas alone, you also have to take into account the relativity of simultaneity. See my post here for a numerical example of how to take into account all three factors to show that two frames will both measure a light beam to move at c.
Grimble said:
If we say that v = 0.866c then [itex] \gamma = 2 [/itex] and if we apply this to our scenario above
we find that the time for light to traverse the diagonal path = [itex]\gamma[/itex] = 2 seconds inertial time
Again I'm confused--wasn't inertial time supposed to be time in the clock's own rest frame? Why would you use gamma to find that time? If the mirrors are 1 light-second apart, the time in the clock's rest frame will be 1 second, naturally. I suppose you're free to imagine that the mirrors are gamma light-seconds apart in the clock's rest frame, but there's no reason why relativity demands this, the mirrors can be set to any distance apart you wish in the clock's rest frame.
Grimble said:
= [itex]2\gamma[/itex] (or [itex]{\gamma^2}[/itex]) [itex] = 4 [/itex]seconds transformed time
and the corresponding diagonal distance will be 4 transformed light seconds
Yes, if the vertical distance between the mirrors happens to be 2 light seconds, and the mirrors are moving at 0.866c in the observer's frame, then the observer will see the diagonal distance as 4 light seconds and the time between ticks as 4 seconds. But again there's no reason why the vertical distance had to be 2 light seconds, you could have equally well said it was some other arbitrary distance like 3.5 light seconds, in which case the time between ticks in the observer's frame would be 2*3.5 = 7 seconds.
Grimble said:
so the observer observing the clocks reference frame will see the light take 4 transformed seconds to cover the 4 transformed light seconds; that is the equivalent of 2 seconds to cover 2 lightseconds inertial time. (all at the speed of c)
The observer isn't "observing the clocks reference frame", he's just observing the clock, and measuring it from the perspective of his own inertial frame. He doesn't have to know anything about the Lorentz transformation whatsoever, he can just measure the time between ticks directly using synchronized clocks at rest in his own frame (or else he can figure out what the time between ticks must be given the vertical distance between the mirrors, the speed at which the clock is traveling which allows him to calculate the diagonal distance using the Pythagorean theorem, and the assumption based on the second postulate that the light must move at c in his frame). If he uses a network of synchronized clocks at rest in his frame, then he just has to note the time T1 on a clock of his that was right next to the bottom mirror when the light was emitted ('right next to' so that he is assigning times using only local measurements and he doesn't have to worry about delays between when an event happens and when a signal from the event reaches one of his clocks), and then note the time T2 on a clock of his that was right next to the top mirror when the light reached it, and the time between ticks in the observer's frame will just be T2 - T1. If the two mirrors happen to be 2 light-seconds apart vertically, and the light clock is moving at 0.866c, then T2 - T1 will equal 4 seconds as you say, but you can see that there was no need for the observer to make use of the Lorentz transformation to find this time (although of course it will agree with the time predicted by the Lorentz transformation if you start with the time in the clock's rest frame and then transform into the observer's frame).
 
Last edited:
  • #128
I've just had a go at making a spacetime diagram of JesseM's example, in post #9 there. This is a traditional Minkowski spacetime diagram in which two inertial frames of reference, with a uniform relative velocity, are superimposed for comparison.

I haven't written c in the calculations because Jesse used units of seconds and light seconds, in which the speed of light is 1 light second per second. 1 light second is a unit of length, defined as how far light travels in 1 second. It's approximately 3 * 10^8 metres, about 3/5 of the way from the Earth to the moon. Using normalised units such as this makes the equations simpler.

Events which lie on the same horizontal line (i.e. any line parallel with the horizontal x axis) with each other are simultaneous in Jesse's rest frame (although not simultaneous in the moving ruler's rest frame).

Events which lie on the same vertical line (i.e. any line parallel with the vertical t(ime) axis) happen in the same place with respect to Jesse's rest frame.

The parallel lines marked with a single slash are the world lines of the ends of what Jesse calls the "moving ruler" (the world lines of the two clocks). An object's world line is its trajectory through spacetime; the world line shows the object's location in space at every instant in time. Because the ends of the moving ruler are, by definition, not moving in the ruler's rest frame, events which happen on one of these lines have the same spatial coordinates (in this ruler's rest frame) as any other events on the same line.

Lines with a double slash are lines of simultaneity in the rest frame of the moving ruler. An event which lies on one of those lines is simultaneous in the moving ruler's rest frame with all other events which lie on that line (although not simultaneous in Jesse's rest frame). Such events have the same time coordinate in the moving ruler's rest frame.

The line labelled "light cone" is the world line of the light. There are two significant events on this world line. The first is the emission of the flash of light from the common spacetime origin (where and when the zero end of the moving ruler coincides with Jesse's). The second is the arrival of the light at the other end of the moving ruler. In the moving ruler's rest frame, this second event is 50 light seconds away from the origin, and happens 50 seconds later in time (according to both of the clocks, as they're synchronised in that frame). In Jesse's rest frame, the second event occurs 100 light seconds away from first and happens 100 seconds later in time. Thus the speed of light is the same in both frames.

Various relevant values are shown in terms of gamma = 1/sqrt(1 - (v/c)^2), which in this case is 1/sqrt(1 - (3/5)^2) = 5/4 = 1.25. I've also shown alternative, equivalent way of calculating these values, using the hyperbolic functions cosh (hyperblic cosine), sinh (hyperbolic sine) and tanh (hyperbolic tangent).

Two values which Jesse didn't mention are the 37.5 seconds and 62.5 light seconds. These are respectively the t and x coordinates in Jesse's rest frame of the event of the clock at the moving ruler's far end showing time = 0. In the ruler's rest frame, this event happens simultaneously with the the event of both clocks showing 0 at the shared spacetime origin (and in the ruler's rest frame therefore has no time component, only a space component of 50 light seconds), but in Jesse's rest frame, as he stands of the spacetime origin, it still lies 37.5 seconds in his future!

I used a slightly different colour for some of the labels round the edge. This has no special significance; it was just to separate them more clearly from labels next to them.

I hope I've got everything right. All criticism welcome!
 

Attachments

  • Minkowski-diagram.jpg
    Minkowski-diagram.jpg
    43.1 KB · Views: 420
  • #129
Grimble said:
[...] an identical clock in the oberver's frame of reference [...]

Are you clear on the fact that if there are two clocks, each moving with some contant velocity relative to the other, both clocks are in all frames of reference, in the sense that both can be described using the language of special relativity with respect to any inertial frame of reference? I think what you have in mind here is "an identical clock which is at rest in the observer's rest frame" (i.e. an identical clock at rest with respect to this observer, not moving with respect to the observer, at a constant distance from the observer--whether located in the same place as the observer or in any other place, so long as there are no tidal effects of gravity present there).
 
Last edited:
  • #130
Rasalhague said:
Are you clear on the fact that if there are two clocks, each moving with some contant velocity relative to the other, both clocks are in all frames of reference, in the sense that both can be described using the language of special relativity with respect to any inertial frame of reference? I think what you have in mind here is "an identical clock which is at rest in the observer's rest frame" (i.e. an identical clock at rest with respect to this observer, not moving with respect to the observer, at a constant distance from the observer--whether located in the same place as the observer or in any other place, so long as there are no tidal effects of gravity present there).

Yes, of course, and thank you, I was being a little slipshod in my language there

Grimble:redface:
 
  • #131
Thank you, JesseM, I can see from your reply that we are understanding different things from what I write. Let me take your input and see if by, applying what you have pointed out I can re-write my thoughts so that you can understand what I am saying:smile:

Let me start by defining the terms I use.
Firstly I do not use the terms primed and unprimed as I have seen these used both ways round and swapped so many times that their use, for me at least, has been compromised;

Inertial units (time and space) are those measured locally (by a local observer within that same frame of reference) in any inertial frame of reference. That they are the same in all inertial frames of reference (Galilean frames) is a demand of the first postulate.

Transformed units (time and space) are those same inertial units, transformed using the Lorentz equations, which is how they will appear when observed from another inertial frame of reference and are a function of their relative velocity.
(I am not saying that one is the clock and the other is the observer, I am saying that those terms are dependent on where they are measured from. They are two separate and distinct scales of measurement that are applied where appropriate)
In order to make it easier to follow I will use the subscripts 'i' and 't' applied to the terms that denote measurements to indicate the units that they are measured in.

JesseM said:
Again, do you understand the difference between talking about an object moving relative to you and talking about a frame moving relative to you? If I am at rest on the embankment, I can perfectly well analyze the behavior of a moving train, or a moving light clock aboard the train, using only the embankment frame, without ever making reference to the train's own rest frame--do you disagree with that?

Not at all,

If not, note that this is in fact the sort of thing I did with the moving light clock, figuring out the time between ticks using only the observer's frame, not the light clock's own rest frame.

and Galileo and Newton would have agreed that the increased distance traveled by the light would have meant that the speed of light would have increased but the time would have remained constant (relativity principle = 1st postulate) and they would have agreed that the time would have had to increase if the speed were to remain constant (light speed principle = 2nd postulate)
But Einstein would have said No! We must comply with Both Postulates.

Let me address the previous post you quote.

We have the horizontal distance = vT,
The vertical distance h (which, if the clock is ticking seconds in this frame = 1 x c)
and the diagonal distance D, where [tex]D = \sqrt{{v^2}{T^2} + {h^2}}[/tex]

And you quite rightly say that only one frame is needed to reach this point. In fact it could be done using either frame.
Consider, if you will, that if we were to use the light clock's own frame of reference, then we should still say that the observer, traveling at v relative to the clock would still see the diagonal [tex]D = \sqrt{{v^2}{T^2} + {h^2}}[/tex].

I have no problem with this at all, nor with the conclusion that:
T is the time for light to travel from the mirror to the observer while
t is the time for the light to travel back to the source
and that if the clock is ticking seconds in its own frame of reference then it will take [itex]γ = \frac{1}{\sqrt{1-\frac{v^2}{c^2}}}[/itex] seconds, measured in that same frame, to reach the observer. . . . . . .(1)

But the first postulate also requires that an identical clock, stationary in the observer's frame of reference would also be ticking seconds, identical seconds, as they are both inertial frames of reference, or Galilean frames as Einstein termed them. . . . . . .(2)

What we have to determine here, is how to resolve the difference in what the observer sees, between the 1 second that his own identical clock takes to tick and the γ seconds that he observes the moving clock take for each tick.

You have it backwards here. The time in the observer's frame will be the greater time, not the lesser time (and the gamma factor γ is always greater than 1). So, if the time in the clock's frame is 1 second, the time in the observer's frame will be gamma seconds (and if the time in the clock's frame is T seconds, the time in the observer's frame is T*gamma seconds)

But we have just established those times in (1),(2) above!

Thankfully Einstein gave us the way to resolve the difference in what the observer sees,:
We transform the time observed in the clock's frame using the Lorentz Transformation, and thus we find that T, in transformed units is equal to 1 second inertial time. . . . . . (3)
(And remember, I am not saying that one is the clock and the other is the observer, I am saying that those terms are dependent on where they are measured from. They are two separate and distinct scales of measurement that are applied where appropriate)
Yes, indeed, one could take the time from any inertial frame (they are all, by definition equal, after all) and transform it (transformed time being that in one frame viewed from another).

But, in the initial scenario and using only one frame, Ti seconds = γti seconds in time( where γ or [itex]\frac{1}{1-\frac{v^2}{c^2}}[/itex] is the ratio of the distance between the mirrors and that from the mirror to the observer, the diagonal distance), then
[itex]{T_t} = \gamma {t_t}[/itex] seconds in transformed time ...by just changing the units
and as we shewed above in (3)
[itex]{T_t} = {t_i}[/itex] seconds ininertial time (and t, one tick of our clock, = 1)
therefore
γ seconds in transformed time = 1 second in inertial time
i.e. γ is the conversion factor between inertial and transformed units.

So, to reiterate, 1 second inertial time (the observer's own clock) is equal to the time T for the moving clock which is in transformed units and, therefore, to γt seconds transformed units.
But T = 1 giving 1 second inertial time = γ seconds transformed time

or in the more usual terms used [tex]{t^'} = \frac{t}{\gamma}[/tex]

If t is supposed to be the time in the clock's own rest frame and t' is the time in the observer's frame, then this formula is wrong, you should be multiplying by gamma rather than dividing by it: {itex] {t^'} = t*\gamma[/itex].

I have seen the time dilation formula written (and used) in both forms:

1) [tex]{t^'} = t * \gamma[/tex]

2) [tex]{t^'} = \frac{t}{\gamma}[/tex]

1) is the more commonly used but my research indicates that it is 2) that is the correct one for the following reasons:
  1. It is the one Einstein formulated;
  2. It is the one derived from the Lorentz equations;
  3. The 'light clock' derivation is incomplete (as I am shewing in this thread) and reverses the correct derivation;
  4. A http://www.answers.com/topic/special-relativity#Time_dilation_and_length_contraction" reverses the terms and thereby reverses the derived formula;
  5. Einstein himself, proved
    "[URL="[PLAIN]http://www.bartleby.com/173/11.html"[/URL] that [itex]{x} = c{t}[/itex] and that [itex]{x^'} = c{t^'}[/itex] so the formulae for Length contraction and time dilation have to be analagous or these two equations cannot both be correct;
  6. Minkowski space time shews quite conclusively exactly how length contraction and time dilation are, in fact, the same process and could, incidentally, have been termed 'length dilation' and 'time contraction' and all those terms would have been correct! (I will shew this later).

which is what we should expect as it reflects the analagous formula for length contraction [tex]{L^'} = \frac{L}{\gamma}[/tex]

This is the correct formula for length contraction if L is the object's length in its own rest frame and L' is the length in the observer's frame, but you can see that it's not exactly analogous to the correct formula for time dilation I wrote above, the formula for time dilation involves multiplying by gamma while the formula for length contraction involves dividing by gamma.
But no it doesn't, just because someone at sometime in the distant past decided to use the labels length-contraction and time-dilation implying one increases while the other decreases people have been making the mistake of thinking the formulae are opposites, while they are in fact analagous.

from which we can see that if [tex]c = \frac{L}{t}[/tex] then [tex]c = \frac{L^'}{t^'}[/tex] which of course it has to do.

Nope, the fact that the speed of light is c in both frames cannot be derived from the length contraction and time dilation formulas alone, you also have to take into account the relativity of simultaneity. See my post here for a numerical example of how to take into account all three factors to show that two frames will both measure a light beam to move at c.

Yet Einstein did it directly from the Lorentz transformation formula, very simply. It isn't complicated. See paragraph 6

Grimble:smile::smile::smile:
 
Last edited by a moderator:
  • #132
Grimble said:
Inertial units (time and space) are those measured locally (by a local observer within that same frame of reference) in any inertial frame of reference. That they are the same in all inertial frames of reference (Galilean frames) is a demand of the first postulate.

What do you mean by "local"? The "same frame of reference" as what? What you you mean by "in the same frame of reference [...] in any inertial frame of reference"?

The first postulate, in the form Einstein originally presented it, says only that the laws by which the states of physical systems change don't depend on which of two frames, in uniform translatory motion relative to one another, these changes of state are referred to. There's no mention here of either frame being valid only at some location or in some region. It may be necessary for practical purposes to limit the scope of a frame to a specified region of spacetime, small enough for the tidal effects of gravity to be undetectable to whatever instruments are available, but these thought experiments used to introduce the concepts of special relativity leave aside such practicalities and assume that there's no significant gravity. They do this to illustrate effects due only to differences in relative motion of inertial reference frames. In Minkowski spacetime, there's no limit to the size of a reference frame. All frames extend through all spacetime, but the time and space components of vectors are certainly not the same in all inertial frames; they're different in frames moving at different relative velocities. What stays the same is the magnitude of vectors, such as the spacetime interval, and the physical laws themselves. It's only by acknowledging that the individual components of vectors change that the laws themselves can comply with this postulate.

Grimble said:
Transformed units (time and space) are those same inertial units, transformed using the Lorentz equations, which is how they will appear when observed from another inertial frame of reference and are a function of their relative velocity.

Do you mean to distinguish between input (source) and output (target) of the transformation? When the space and time components of vectors are changed by the Lorentz transformation, the new space and time components are simply the space and time components of the vector referred to another, equally valid, inertial reference frame. All frames involved here are inertial: both input and output. For this reason, I find the distinction "inertial" versus "transformed" (which you admit is also inertial) confusing.

Grimble said:
(I am not saying that one is the clock and the other is the observer, I am saying that those terms are dependent on where they are measured from.

The value of the Lorentz transformation is not dependent on where the inputs are measured from! You could picture each frame, as Taylor and Wheeler do, as an infinite grid of clocks connected by meter-long bars. One frame passes through another, but they occupy the same space. Or you could imagine a single observer recording events at a distance, in which case they'd have to take into account the time it takes for signals to reach them. The Lorentz transformation deals with the differences in time and space as referred to inertial reference frames which differ only in that they're moving at constant velocity in different directions. It deals with the differences that remain after signalling delays and other precticalities have been accounted for.
 
Last edited:
  • #133
Inertial units (time and space) are those measured locally (by a local observer within that same frame of reference) in any inertial frame of reference. That they are the same in all inertial frames of reference (Galilean frames) is a demand of the first postulate.

Rasalhague said:
What do you mean by "local"? The "same frame of reference" as what? What you you mean by "in the same frame of reference [...] in any inertial frame of reference"?

I think that what I wrote was quite clear - (by a local observer within THAT same frame of reference)

I was defining what I meant by the terms I used.

By inertial units I mean units within an inertial frame of reference, as measured and referred to within that same frame of reference, in order to avoid anyone claiming that they could be anything else.

By transformed units I mean units that have been transformed by Lorentz transformations.
 
  • #134
Grimble said:
Let me start by defining the terms I use.
Firstly I do not use the terms primed and unprimed as I have seen these used both ways round and swapped so many times that their use, for me at least, has been compromised;

Inertial units (time and space) are those measured locally (by a local observer within that same frame of reference) in any inertial frame of reference. That they are the same in all inertial frames of reference (Galilean frames) is a demand of the first postulate.
I don't get it. When you say "within the same frame of reference", "same" as what, exactly? And if they are measured relative to a particular frame, then why do you say "they are the same in all inertial frames"?

I think it would help if we dealt with a particular example. Suppose we have two observers, A and B, who are moving inertially relative to one another. Each observer has a ruler at rest relative to themselves, and at each marking on their own ruler there is attached a clock, which is naturally also at rest relative to that observer since the ruler-marking is at rest. Also suppose the different clocks on a given observer's ruler have been synchronized in that observer's frame (because of the relativity of simultaneity, this means each observer will say the clocks on the other observer's ruler are out-of-sync). Each observer uses their own ruler/clock system to make local measurements of the coordinates of events, by looking at which ruler-marking and clock was right next to the event when it happened. For example, if observer A looks through his telescope and sees an explosion happening in the distance, then if he sees the explosion happened right next to the 15-light-second mark on his ruler, and sees that the clock at the 15-light-second mark read 10 seconds at the moment the explosion was happening, then he assigns that event coordinates (x=15 light seconds, t=10 seconds) in his own inertial frame.

If you look at my thread an illustration of relativity with rulers and clocks you can see some diagrams showing two such ruler/clock systems of different observers moving right alongside each other, drawn from the perspective of two different frames. The different frames disagree about which ruler's markings are shrunk and which set of clocks are running slower (and which are synchronized and which are out-of-sync), but they always agree on which readings locally coincide. For example, in the top part of this diagram we see a diagram drawn from the perspective of the A frame, and in the bottom part is a diagram drawn from the perspective of the B frame, although many aspects of the diagrams look different, the circles show that if you pick a particular local event than both frames agree:

MatchingClocks.gif


For example, suppose a red firecracker explodes next to the 346.2 meter mark on A's ruler, when A's clock at that mark reads 1 microsecond; then the diagram shows that this firecracker explosion must have also been next to the 173.1 meter mark on B's ruler, when B's clock there read 0 microseconds. Likewise, if you look at some of the earlier diagrams on that thread, you can see that if a blue firecracker exploded next to the 0 meter mark on A's ruler, when A's clock there read 0 microseconds, then the blue firecracker must also have exploded next to the 0 meter mark on B's ruler, when B's clock there read 0 microseconds. No one will disagree about local facts like this.

However, if we ask about the distance and time between the explosion of the blue firecracker and the explosion of the red firecracker, then this is a frame-dependent question. In the A frame the blue firecracker explosion had coordinates (x=0 meters, t=0 microseconds) and the red firecracker explosion had coordinates (x=346.2 meters, t=1 microseconds), so in A's frame the distance between these events was 346.2 meters and the time between them was 1 microsecond. On the other hand, in the B frame the blue firecracker explosion had coordinates (x'=0 meters, t'=0 microseconds) while the red firecracker explosion had coordinates (x'=173.1 meters, t'=0 microseconds), so in B's frame the distance between these events was 173.1 meters and the time between them was 0 microseconds (in B's frame they were simultaneous).

So, would A's measurement of a distance of 346.2 meters and a time of 1 microsecond between the explosions be a measurement in "inertial units" in your terminology? It was after all based on local measurements on A's inertial ruler/clock system. But when B uses his own local measurements on his own inertial ruler/clock system, he gets a different answer for the distance and time between these two explosions (although he does not disagree about which marking and clock-reading on A's system were next to the explosions). Would B's measurement of a distance of 173.1 meters and a time of 0 microseconds also be a measurement in "inertial units"? If so, when you said that inertial units are the same in all inertial frames, what did you mean?
Grimble said:
That they are the same in all inertial frames of reference (Galilean frames) is a demand of the first postulate.
The first postulate obviously doesn't demand that the distance and time between a given pair of events be the same when different inertial observers measure it using their own ruler/clock systems. If that's not what you meant, then what are you saying would be the same in all inertial frames of reference?
Grimble said:
Transformed units (time and space) are those same inertial units, transformed using the Lorentz equations, which is how they will appear when observed from another inertial frame of reference and are a function of their relative velocity.
Again, can you explain how this terminology applies to my above example? It's true, for example, that if you know that the A frame assigned the red firecracker explosion coordinates (x=346.2 meters, t=1 microsecond), then if you just plug these coordinates into the Lorentz transformation (with gamma = 2 and v = 0.866c), you can deduce that the B frame will assign the red firecracker explosion coordinates (x'=173.1 meters, t'=0 microseconds). Does this make the latter set of coordinates "transformed units", even though they are just what B found using his own inertial ruler/clock system? And note that of course you can also go in reverse--if at first you only know that B assigned the red firecracker explosion coordinates (x'=173.1 meters, t'=0 microseconds), then you can apply the Lorentz transformation to that to deduce that A assigned this same explosion the coordinates (x=346.2 meters, t=1 microsecond). So can every measurement be seen as both inertial units and transformed units, depending on what data you start with and then apply the Lorentz transformation to? If not, then again, please explain the difference between "inertial units" and "transformed units" in terms of the example I have given with the firecrackers and the two inertial ruler/clock systems.
Grimble said:
and Galileo and Newton would have agreed that the increased distance traveled by the light would have meant that the speed of light would have increased but the time would have remained constant (relativity principle = 1st postulate)
What does that have to do with the first postulate? The first postulate in no way demands that the time between the events (light hitting bottom mirror) and (light hitting top mirror) be the same in both frames (just like it didn't demand that the time between the blue and red firecracker explosions in my example above should be the same in both frames), if it did then relativity would violate the first postulate. The first postulate just demands that the laws of physics obey the same equations in both frames.
Grimble said:
and they would have agreed that the time would have had to increase if the speed were to remain constant (light speed principle = 2nd postulate)
Here you are speculating about what Newton and Galileo would have said about physics that didn't come along until well after they were dead. When I talked about what Newton and Galileo would have said, I didn't mean to talk about what they might have said if they had lived to see new ideas long after their time, I just meant to talk about what is true in classical pre-relativistic physics. In classical physics the 2nd postulate is just false, you can't have any object that has the same speed in all inertial frames.
Grimble said:
But Einstein would have said No! We must comply with Both Postulates.
If you are somehow under the impression that the first postulate says different frames should agree on the time and distance between events, and that Einstein says we should comply with that, you are badly misunderstanding the meaning of the first postulate, which again is just about the general equations for the laws of physics, not about the time and distances between a specific pair of events. In fact even in classical physics the distance between a pair of events can be different in two different inertial frames, although in classical physics (unlike in relativity) the time between a pair of events is the same in all inertial frames.
Grimble said:
We have the horizontal distance = vT,
The vertical distance h (which, if the clock is ticking seconds in this frame = 1 x c)
and the diagonal distance D, where [tex]D = \sqrt{{v^2}{T^2} + {h^2}}[/tex]

And you quite rightly say that only one frame is needed to reach this point. In fact it could be done using either frame.
Yes, although of course in the rest frame of the clock, v=0 so the path from one mirror to another is purely vertical rather than diagonal in this frame.
Grimble said:
Consider, if you will, that if we were to use the light clock's own frame of reference, then we should still say that the observer, traveling at v relative to the clock would still see the diagonal [tex]D = \sqrt{{v^2}{T^2} + {h^2}}[/tex].
You seem to be confused about what physicists mean when they talk about "using" a given frame of reference--it means that you analyze things using only the distance and time coordinates of that frame (along with coordinate-invariant things like proper times and statements about pairs of events that locally coincide), and don't refer to the coordinates of any other frame. So, it's an incorrect usage of the lingo to say that you can "use the light clock's own frame of reference" to deduce what coordinate distance the light traveled in the observer's frame (though you can use the light clock's frame to figure out what markings and clock readings on the observer's ruler/clock system would line up with the events of the light hitting the bottom and top mirrors).
Grimble said:
I have no problem with this at all, nor with the conclusion that:
T is the time for light to travel from the mirror to the observer while
t is the time for the light to travel back to the source
and that if the clock is ticking seconds in its own frame of reference then it will take [itex]γ = \frac{1}{\sqrt{1-\frac{v^2}{c^2}}}[/itex] seconds, measured in that same frame, to reach the observer. . . . . . .(1)
Huh? What do you mean when you say "it will take [itex]\gamma = \frac{1}{\sqrt{1-\frac{v^2}{c^2}}}[/itex] seconds, measured in that same frame, to reach the observer"? First of all, the time dilation formula [tex]t' = t*\sqrt{1 - v^2/c^2}[/tex] is normally understood purely in terms of relating the time between ticks in the clock's rest frame to the time between ticks in the frame of an observer moving relative to the clock, the idea that it should have something to do with the time for the light of a tick to reach an observer moving relative to the clock as measured in the clock's own rest frame appears to be an idea unique to you. Second, it's not even clear what you mean when you talk about the time for it to reach an observer--won't this be totally dependent on how far the observer is from the clock? If the observer is right next to the source at the bottom mirror at the moment the light has traveled back to the source from the top mirror, then if it takes 1 second for the light to travel from the source to the top mirror and back to the source, that must mean it also takes 1 second for the light to travel from the source to the top mirror and down to the observer in this frame (of course if we knew the time on the observer's clock when the light left the source according to this frame's definition of simultaneity, we could use this frame to calculate the time on the observer's clock when the light returns to the source, and it might be different than 1 second--is this the sort of thing you were getting at?)
Grimble said:
But the first postulate also requires that an identical clock, stationary in the observer's frame of reference would also be ticking seconds, identical seconds, as they are both inertial frames of reference, or Galilean frames as Einstein termed them. . . . . . .(2)

What we have to determine here, is how to resolve the difference in what the observer sees, between the 1 second that his own identical clock takes to tick and the γ seconds that he observes the moving clock take for each tick.
Here you seem to be saying that the γ seconds is supposed to be the time in the observer's own frame, but before you said that "if the clock is ticking seconds in its own frame of reference then it will take [itex]\gamma = \frac{1}{\sqrt{1-\frac{v^2}{c^2}}}[/itex] seconds, measured in that same frame, to reach the observer". Is γ seconds supposed to be the time between two events in the clock's own frame, or the time between two events in the observer's frame?
Grimble said:
Thankfully Einstein gave us the way to resolve the difference in what the observer sees,:
We transform the time observed in the clock's frame using the Lorentz Transformation, and thus we find that T, in transformed units is equal to 1 second inertial time. . . . . . (3)
Please consider that your understanding of SR and what Einstein meant may just be badly confused. Einstein never introduced any distinction between "inertial time" and "transformed time", in SR we just talk about times in different inertial frames, and the Lorentz transformation just distance and time intervals (or distance and time coordinates of individual events) of one inertial frame to the intervals between the same events in a different inertial frame. Also, in SR there is no notion that the first postulate demands that the time and distance intervals between a given pair of events be the same in different inertial frames.
Grimble said:
Yes, indeed, one could take the time from any inertial frame (they are all, by definition equal, after all) and transform it (transformed time being that in one frame viewed from another).
The notion of one frame "viewing" another also is not part of SR. Each frame is used to analyze things in terms of the coordinates of that frame alone, and then the Lorentz transformation relates an "analysis-wholly-in-frame-A" to an "analysis-wholly-in-frame-B". For example, in my example above with two firecracker explosions, the wholly-in-frame-A analysis gives the distance between them as 346.2 meters and the time between them as 1 microsecond, while the wholly-in-frame B analysis gives the distance between them as 173.1 meters and the time between them as 0 microseconds. If we start out knowing the wholly-in-frame-A intervals, we can plug them into the Lorentz transformation to deduce the wholly-in-frame-B intervals like so:

gamma*(x - vt) = 2*(346.2 meters - 0.86603c*1 microsecond) = 2*(346.2 meters - (259.63 meters/microsecond)*1 microsecond) = 173.1 meters

gamma*(t - vx/c^2) = 2*(1 microsecond - v*(346.2 meters)/c^2) = 2*(1 microsecond - (259.63 meters/microsecond)*(346.2 meters)/(299.79 meters/microsecond)^2) = 2*(1 microsecond - 1 microsecond) = 0 microseconds

Likewise, if we start out knowing the wholly-in-frame B intervals, we can plug them into the Lorentz transformation to deduce the wholly-in-frame-A intervals:

gamma*(x' + vt') = 2*(173.1 meters - 0.86603c*0 microseconds) = 2*173.1 meters = 346.2 meters

gamma*(t' + vx'/c^2) = 2*(0 microseconds + v*(173.1 meters)/c^2) = 2*(0 microseconds + (259.63 meters/microsecond)*(173.1 meters)/(299.79 meters/microsecond)^2) = 2*(0.5 microseconds) = 1 microseconds
Grimble said:
But, in the initial scenario and using only one frame, Ti seconds = γti seconds in time( where γ or [itex]\frac{1}{1-\frac{v^2}{c^2}}[/itex] is the ratio of the distance between the mirrors and that from the mirror to the observer, the diagonal distance)
Again I see no reason why the gamma formula would have anything to do with the "distance to the observer" in the clock's rest frame, you're either confused about what these equations mean in ordinary SR or you're trying to introduce your own novel ideas which are not part of mainstream SR. And the observer's distance from the mirror (which one? Top or bottom?) is constantly changing in this frame, so what moment do you want to talk about the distance from observer to mirror, anyway?
Grimble said:
then
[itex]{T_t} = \gamma {t_t}[/itex] seconds in transformed time ...
Again, nothing in mainstream SR corresponds to your distinction between "inertial" time and "transformed" time as far as I can tell, are you trying to introduce new ideas here or are you under the impression that what you are saying is a part of regular SR? Either way you haven't clearly explained what "inertial time" and "transformed time" are supposed to mean, please show how these terms would apply to a specific numerical example like my example with the two firecracker explosions.
 
  • #135
(continued from previous post)

Grimble said:
I have seen the time dilation formula written (and used) in both forms:

1) [tex]{t^'} = t * \gamma[/tex]

2) [tex]{t^'} = \frac{t}{\gamma}[/tex]

1) is the more commonly used but my research indicates that it is 2) that is the correct one for the following reasons
If different people write it differently it's just because their definition of the primed and unprimed frame is different. If unprimed is the time between a pair of events (like the ticks of a clock) in the frame where those events happen at the same position in space (as would be true in the clock's rest frame if the events represent ticks of a clock), and primed is the time between the same pair of events in a frame moving at speed v relative to the first frame, then the correct formula is 1) above--in the case of clock ticks, the time is greater in the frame where the clock is moving than in the clock's rest frame. On the other hand, if you use primed to be the time in the frame where the events happen at the same position (like the clock's rest frame), and unprimed to represent the time in the frame moving at v relative to the first, then 2) would be the correct formula.
Grimble said:
  1. It is the one Einstein formulated;
  2. It is the one derived from the Lorentz equations;

  1. What derivation are you thinking of? Again, it all depends on which frame is supposed to be the one where the events happen at the same position. If we have unprimed be the one where they're at the same position, and we arbitrarily align the origin with the first event so that it has coordinates (x=0, t=0), then the second event must have coordinates of the form (x=0, t=T), where the constant T can have whatever value we choose. Then the Lorentz transformation is:

    x' = gamma*(x - vt)
    t' = gamma*(t - vx/c^2)

    If we plug (x=0, t=0) into this we get the coordinates of the first event in the primed frame as x'=0, t'=0. Then if we plug (x=0, t=T) in we get:

    x' = gamma*(-vT)
    t' = gamma*(T)

    So the second event has time coordinate t'=gamma*T in the primed frame, and since the first event had t'=0 in the primed frame, in the primed frame the time interval T' between the two events must be T' = gamma*T, same as your formula 1) above.

    On the other hand, if we assume the primed frame was the one where they happened at the same position, then since the primed frame moves at speed v from the perspective of the unprimed frame, in time T the clock will have moved a distance vT, so if the first tick happened at (x=0, t=0), the second must have happened at (x=vT, t=T). The first event again must have coordinates (x'=0, t'=0) in the primed frame when we use the Lorentz transformation. But the second event will have:

    x' = gamma*(vT - vT) = 0
    t' = gamma*(T - v^2*T/c^2) = gamma*T*(1 - v^2/c^2) = T*(1 - v^2/c^2)/sqrt(1 - v^2/c^2) = T*sqrt(1 - v^2/c^2) = T/gamma.

    So, in the primed frame the time interval T' between the two events must be T' = T/gamma in this case. Again, it's just a matter of whether you want primed or unprimed to be the frame where the events happened at the same position, once you have established that convention, there is no question about what the time dilation formula is supposed to look like.
    Grimble said:
    [*]A http://www.answers.com/topic/special-relativity#Time_dilation_and_length_contraction" reverses the terms and thereby reverses the derived formula;
    How does it reverse the terms? If follows the more common convention where unprimed is the rest frame of the clock, and therefore derives your formula 1), T' = gamma*T.
    Grimble said:
    [*]Einstein himself, proved
    "[URL="[PLAIN]http://www.bartleby.com/173/11.html"[/URL] that [itex]{x} = c{t}[/itex] and that [itex]{x^'} = c{t^'}[/itex] so the formulae for Length contraction and time dilation have to be analagous or these two equations cannot both be correct;
    He did not show that x = ct and x'=ct' are general relations which hold for events of arbitrary coordinates, this was just supposed to be the equation of motion for a light beam which was released from the x=0 at t=0 (which also is assumed to coincide with x'=0 and t'=0 in the primed frame in the Lorentz transformation). It's easy to show using the Lorentz transformation that if you pick an event on the worldline of this light beam which occurs at coordinates x=cT and t=T in the unprimed frame (which satisfies x = ct), then in the primed frame this same event has coordinates:

    x' = gamma*(x - vt) = gamma*(cT - vT) = gamma*T*(c - v) = c*gamma*T*(1 - v/c)
    t' = gamma*(t - vx/c^2) = gamma*(T - vcT/c^2) = gamma*T*(1 - v/c)

    So, you can see from this that is true that this event must have coordinates which satisfy x'=c*t' in the primed frame.

    And I already showed that both version 1) and version 2) of the time dilation formula can be derived from the Lorentz transformation depending on which frame you want the clock to be at rest in, so you can see that both formulas must be equally compatible with the equation of motion for the light beam, since they were all derived from the same Lorentz transformation equations.
    Grimble said:
    [*]Minkowski space time shews quite conclusively exactly how length contraction and time dilation are, in fact, the same process and could, incidentally, have been termed 'length dilation' and 'time contraction' and all those terms would have been correct! (I will shew this later).
    Time dilation and length contraction are conceptually different things when illustrated on a Minkowski diagram--time dilation deals with the time between a single pair of events in two different frames, while length contraction does not deal with the distance between a single events in two different frames, rather it deals with the distance between two parallel worldlines at a single moment in two different frames (with 'at a single moment' depending on each frame's definition of simultaneity). It would be possible to come up with a spatial analogue for time dilation which deals with the distance between a single pair of events, in which case the equation would look just like the time dilation equation, and likewise to come up with a temporal analogue for length contraction which deals with the time between two parallel spacelike surfaces at a single position in two different frames, in which case the equation would look just like the length contraction equation. If you're interested you can take a look at the diagram I drew which neopolitan posted in post #5 of this thread, where we were discussing the issue of whether it's meaningful to talk about "time contraction" or "length dilation" (I don't really recommend reading the whole thread though, it went on a lot of tangents).
    Grimble said:
    But no it doesn't, just because someone at sometime in the distant past decided to use the labels length-contraction and time-dilation implying one increases while the other decreases people have been making the mistake of thinking the formulae are opposites, while they are in fact analagous.
    Do you agree that if [tex]\Delta t[/tex] represents the time between ticks of a clock in the clock's rest frame, and [tex]\Delta t'[/tex] represents the time between ticks in a frame moving at speed v relative to the clock, then the correct formula is [tex]\Delta t' = \gamma * \Delta t[/tex]? And do you agree that if L represents the distance between either end of an object in the object's rest frame, and L' represents the distance between either end of that object in a frame moving at speed v relative to the object (with the positions of each end measured simultaneously in whatever frame is measuring the distance), then the correct formula is [tex]L' = \frac{L}{\gamma}[/tex]?
    Grimble said:
    from which we can see that if [tex]c = \frac{L}{t}[/tex] then [tex]c = \frac{L^'}{t^'}[/tex] which of course it has to do.
    Nope, you cannot show that the speed of light is the same in two frames using only length contraction and time dilation, you have to take into account the relativity of simultaneity too.
    JesseM said:
    Nope, the fact that the speed of light is c in both frames cannot be derived from the length contraction and time dilation formulas alone, you also have to take into account the relativity of simultaneity. See my post here for a numerical example of how to take into account all three factors to show that two frames will both measure a light beam to move at c.
    Grimble said:
    Yet Einstein did it directly from the Lorentz transformation formula, very simply. It isn't complicated. See paragraph 6
    Obviously you can derive the fact that a light beam moves at c in all frames using the Lorentz transformation, since the Lorentz transformation was itself derived using the second postulate and since time dilation, length contraction and the relativity of simultaneity can all be derived from it too. But we were talking about your claim (which you repeat above) that somehow one could combine the length contraction equation and the time dilation equation alone to get the invariance of c, without making use of the full Lorentz transformation equations, and also without making use of the relativity of simultaneity. It's this claim which doesn't make any sense (on the other hand, if you use the time dilation equation along with what I called the 'spatial analogue of time dilation', which unlike length contraction deals with the distance between a single pair of events, then you can combine these two equations to get the conclusion that distance/time for two events on the worldline of a lightbeam must always equal c).
 
Last edited by a moderator:
  • #136
Grimble said:
I think that what I wrote was quite clear - (by a local observer within THAT same frame of reference)

By local do you mean that the measurements are to be made at all relevant points in spacetime so that there are no delays to take into account between an event and it being recorded? If so, why do you emphasise "within that same frame"? Are you aware that if an observer is present at an event according to one frame, then all frames will agree on the observer being present at that event?

Grimble said:
I was defining what I meant by the terms I used.

By inertial units I mean units within an inertial frame of reference, as measured and referred to within that same frame of reference, in order to avoid anyone claiming that they could be anything else.

By transformed units I mean units that have been transformed by Lorentz transformations.

...these transformed values being intervals of time and space in some other inertial frame, hence my suggestion that terms such as "input" and "output" might be less confusing, or untransformed and transformed, or something like that.

Grimble said:
and Galileo and Newton would have agreed that the increased distance traveled by the light would have meant that the speed of light would have increased but the time would have remained constant (relativity principle = 1st postulate) and they would have agreed that the time would have had to increase if the speed were to remain constant (light speed principle = 2nd postulate)
But Einstein would have said No! We must comply with Both Postulates.

The idea of the time dilation derivation from the example of the light clock is that, since the speed of light is the same in both frames, a greater time must elapse between "ticks" in the frame where the light clock is moving than elapses between ticks in the frame where the light clock is at rest.

Grimble said:
And you quite rightly say that only one frame is needed to reach this point. In fact it could be done using either frame.

In the light clock's rest frame, v = 0, so D = h. The formula is still meaningful, but it doesn't serve its intended purpose of demonstrating how the size of an interval of time between a given pair of events depends on which reference frame they're referred to.

Have a look at the section called Time distortion here which begins "Consider the situation shown in figure f." I often find it helps to read different authors' explanations of the same idea.

http://www.lightandmatter.com/html_books/0sn/ch07/ch07.html

Grimble said:
1) is the more commonly used but my research indicates that it is 2) that is the correct one for the following reasons:

I don't think it's a matter of one being correct and the other incorrect. It just depends what you need to calculate and what you're using the prime symbol to represent.
 
Last edited by a moderator:
  • #137
JesseM said:
I don't get it. When you say "within the same frame of reference", "same" as what, exactly? And if they are measured relative to a particular frame, then why do you say "they are the same in all inertial frames"?.

Before I say anything else will somebody, anybody, please read what I have written?


I DID NOT write "within the same frame of reference" I wrote "within that same frame of reference" - bold used for emphasis!

Which if read as written changes the meaning somwhat.

PS. I apologise Jesse (if I may call you that?) but yours is the second of two replies that have made the same misquote of what I had written...
 
  • #138
JesseM said:
Huh? What do you mean when you say "it will take [itex]\gamma = \frac{1}{\sqrt{1-\frac{v^2}{c^2}}}[/itex] seconds, measured in that same frame, to reach the observer"? First of all, the time dilation formula [tex]t' = t*\sqrt{1 - v^2/c^2}[/tex] is normally understood purely in terms of relating the time between ticks in the clock's rest frame to the time between ticks in the frame of an observer moving relative to the clock, the idea that it should have something to do with the time for the light of a tick to reach an observer moving relative to the clock as measured in the clock's own rest frame appears to be an idea unique to you.

If the clock's time is one second, the height is ct where t=1, the horizontal distance is vt' and the diagonal distance is ct', where t' is the time for the light to reach the observer who passed the clock at the start of the 'tick' all measured in the clock's frame, then [tex]{t^'}= \frac{1}{\sqrt{1-\frac{v^2}{c^2}}}[/tex] by the application of simple old Pythagoras.
 
  • #139
Grimble said:
Before I say anything else will somebody, anybody, please read what I have written?


I DID NOT write "within the same frame of reference" I wrote "within that same frame of reference" - bold used for emphasis!

Which if read as written changes the meaning somwhat.
I don't see how it changes the meaning. I repeat the question, with that word changed:
JesseM said:
Grimble said:
Inertial units (time and space) are those measured locally (by a local observer within that same frame of reference) in any inertial frame of reference. That they are the same in all inertial frames of reference (Galilean frames) is a demand of the first postulate.
I don't get it. When you say "within that same frame of reference", "same" as what, exactly? And if they are measured relative to a particular frame, then why do you say "they are the same in all inertial frames"?
Grimble said:
PS. I apologise Jesse (if I may call you that?) but yours is the second of two replies that have made the same misquote of what I had written...
No problem, and sorry for misquoting (and yeah, feel free to just call me Jesse), but as I said I don't understand why "the same" vs. "that same" makes a difference in how I should interpret your statement. Again, "same" as what?
 
  • #140
Please accept my apologies for the state of this thread, good people, it is so easy to be diverted by arguments of minutiae.

Let me say that I have come to SR on my own using the Einstein paper that I quote from.

I found that to be clear, concise and easy to understand.

I then looked further, on the web, principally in Wikipedia etc. and was intersted to find things that did not match what I had learned from Einstein.

I do not pretend (honestly) to know the answers, but for me, just being told 'thats the way it is' doesn't satisfy, I like to know WHY and HOW.

One of the major problems I find is the constant pulling apart every statement I make and telling me to rephrase it or disecting what the words mean!

I have been told (not just in this thread) that proper time is the term to use and that there is no such thing as proper time...

that there is no need to use A' and B', but that A & B will do: then I am told that I should use A' and B'...

having experienced so much criticism for using the wron terms, I tried defining my own - inertial and transformed units, and earning immediate criticism despite attempting to define exactly what I meant by the use of those terms.

No matter how I try and ask questions or address the points that don't seem to add up for me all I get is constant critical disection of the language I am trying to use.

And it is not just that I am unused to the particular terms you use, but you can't even agree amongst yourselves about the use of the terms.

Top this with a tendency to read into what I am saying, what you expect me to be saying, without bothering, it seems, to actually reading it, and the whole exercise becomes frustrating.

One thing which I find particularly annoying (and which I am sure will annoy anyone who experiences it) is to be told what I am thinking, when what I am told is not, and sometimes is the very opposite, of what I am thinking.

Moaning over!

My background is scientific, I studied physics at university, many years ago, followed by 25 years in computing, where I spent many years solving problems, designing systems and in support work, where the prime skill was to be able to take written documents, designs, and complete software systems and find the bugs in them.

I have come to you for assistance in understanding SR and answering questions that arise where the modern understanding seems to fit uneasily with what Einstein wrote.

So I ask for your patience and your help

Thank you, Grimble:smile::smile::smile::smile::smile:
 

Similar threads

  • Special and General Relativity
Replies
17
Views
2K
  • Special and General Relativity
Replies
16
Views
716
  • Special and General Relativity
Replies
9
Views
382
  • Special and General Relativity
2
Replies
58
Views
2K
  • Special and General Relativity
3
Replies
88
Views
3K
  • Special and General Relativity
Replies
16
Views
1K
  • Special and General Relativity
2
Replies
36
Views
1K
  • Special and General Relativity
Replies
19
Views
1K
  • Special and General Relativity
2
Replies
37
Views
3K
  • Special and General Relativity
2
Replies
45
Views
3K
Back
Top